Answer:
1728
Step-by-step explanation:
1200 borrowed for 2 years at 20% compounded annually ;
Using the compound interest formula :
A = P(1 + r/n)^nt
P = 1200 = principal
Time, t = 2 years
Rate, r = 20% = 0.2
A = 1200(1 + 0.2)^2
A = 1200(1.2)^2
A = 1200 * 1.44
A = 1728
Final amount to be repayed = 1728
In 2017 the Hindu festival of Diwali occurred on Thursday, 19 October. What fraction of the year was this
The Diwali occurred on 0.408 or 40.8% of the year in 2017.
To find the fraction of the year that Diwali occurred in 2017, we need to divide the number of days between Diwali and the start of the year by the total number of days in the year.
Diwali occurred on Thursday, October 19, 2017. The start of the year 2017 is January 1, 2017 which is a Sunday.
There are 365 days in a non-leap year. Since 2017 is not a leap year, there are 365 days in that year.
So the fraction of the year that Diwali occurred in 2017 can be calculated as: (days between Diwali and the start of the year) / (total days in the year) = ( 19 + 31 + 30 + 31 + 30 + 19 ) / 365 = (148) / 365 = 0.408
So, the Diwali occurred on 0.408 or 40.8% of the year in 2017.
Therefore, Diwali occurred on 0.408 or 40.8%
To learn more about fractions,
Visit; brainly.com/question/10354322
#SPJ4
A picture frame can hold a photograph that is 16 inches long and 12 inches wide.
What is the area of the photo that fits the frame?
Answer: 192
Step-by-step explanation: 16*12=192
A car travels 198 miles in 3 hours. Assuming that the distance varies directly with the time, how far will the car travel in 5 hours
Answer:
330 miles
Step-by-step explanation:
198/3 = 66; this shows how fast the car is going per hour
66 * 5 = 330; this is how far the car goes in 5 hours
L 4.6.3 Test (CST): Linear Equations
me.
OA. y+4= -3(x-3)
OB. y-4=-3(x+3)
OC. y-4=3(x+3)
OD. y+4=3(x-3)
(3,-4)
The correct option is OA. y+4= -3(x-3). L 4.6.3 Test (CST): Linear Equations Solution: We are given that a line passes through (3,-4) and has a slope of -3.
We will use point slope form of line to obtain the equation of liney - y1 = m(x - x1).
Plugging in the values, we get,y - (-4) = -3(x - 3).
Simplifying the above expression, we get y + 4 = -3x + 9y = -3x + 9 - 4y = -3x + 5y = -3x + 5.
This equation is in slope intercept form of line where slope is -3 and y-intercept is 5.The above equation is not matching with any of the options given.
Let's try to put the equation in standard form of line,ax + by = c=> 3x + y = 5
Multiplying all the terms by -1,-3x - y = -5
We observe that option (A) satisfies the above equation of line, therefore correct option is OA. y+4= -3(x-3).
Thus, the correct option is OA. y+4= -3(x-3).
For more question on equation
https://brainly.com/question/17145398
#SPJ8
a map is drawn to a scale of 1:500000. the actual distance from Riyadh to Yanbu is 1045 km. calculate the map distance in cm
Answer:
Step-by-step explanation:
Given that 1cm=500000cm
Then using unitary law, we get
Actual distance=2.5cm=2.5×500000=12,50,000cm
What is the sum of a geometric series of 12 terms that begins with 10 and has a common ratio of 2?
Answer:
\(40950\)
Step-by-step explanation:
\(S_n=\frac{a_1(1-r^n)}{1-r}\\\\S_{12}=\frac{10(1-(2)^{12})}{1-2}\\\\S_{12}=\frac{10(1-4096)}{-1}\\\\S_{12}=\frac{10(-4095)}{-1}\\\\S_{12}=10(4095)\\\\S_{12}=40950\)
i need step by step please 3(x-2)^2+1=13
Answer:
\( x_{1} = 0. \: \: \: x_{2} = 4\)
Step-by-step explanation:
\(3(x - 2)^{2} + 1 = 3\)
\(3( {x}^{2} - 4x + 4) + 1 = 13\)
\( {3x}^{2} - 12x + 12 + 1 = 13\)
\( {3x}^{2} - 12x + 13 = 13\)
\( {3x}^{2} - 12x = 0\)
\(x \times (x - 4) = 0\)
\(x = 0\)
\(x - 4 = 0\)
\( \boxed{\green{x_{1} = 0. \: \: \: x_{2} = 4}}\)
what number needs to be added to both sides of the equation in order to complete the square? x2+16x=18
In order to complete the square for the given equation x^2+16x=18, the number added to both sides is 64.
Complete the square is a method used to solve a quadratic equation by changing the form of the equation so that the left side is a perfect square trinomial. To complete the square, add or subtract the same value to both sides. In this case, add the square of half the coefficient of the x -term, (b/2)^2 to both sides of the equation. Hence, the number to add would be (16/2)^2 = 64.
Thus,
x^2+16x + 64 =18 + 64
x^2+16x + 64 =82
x^2+ 2*8x + 64 =82
(x + 8)^2 = 82
Learn more about Complete the square:
https://brainly.com/question/10449635
#SPJ4
The city of London, England, has an
elevation of 11 meters.
Which of these describes the elevation
of London?
below sea level
at sea level
above sea level
Answer:
above sea level
Step-by-step explanation:
Harish wants to buy a smartphone worth Rs 22780.50. But currently he only has Rs 18600 with him. He deposits this amount in a bank which offers 9% simple interest per annum.
How long will it take for the amount to become Rs 22780.50, so that Harish can buy the smartphone?
It will take 5 years for the deposited amount to grow from Rs 18600 to Rs 22780.50, considering a 9% simple interest per annum.
To calculate the time required, we can use the formula for simple interest:
Simple Interest = Principal × Rate × Time
Given that the principal amount (P) is Rs 18600, the rate of interest (R) is 9%, and the target amount (A) is Rs 22780.50, we need to find the time (T).
Rewriting the formula, we have:
Simple Interest = (Principal × Rate × Time) / 100
Since the interest is calculated for the entire amount, the simple interest is equal to the difference between the target amount and the principal:
Simple Interest = A - P
Substituting the values, we have:
A - P = (P × R × T) / 100
Simplifying the equation, we get:
A - P = (18600 × 9 × T) / 100
Now, let's solve for T:
A - P = (167400T) / 100
A - P = 1674T
Substituting the values of A and P, we have:
22780.50 - 18600 = 1674T
Simplifying the equation, we get:
4178.50 = 1674T
Dividing both sides by 1674, we have:
T ≈ 2.496
Since time cannot be in decimal years, we round it up to the nearest whole number. Therefore, it will take approximately 3 years for the amount to become Rs 22780.50, allowing Harish to buy the smartphone.
Learn more about simple interest here
https://brainly.com/question/25793394
#SPJ11
in base $10$, the number $2013$ ends in the digit $3$. in base $9$, on the other hand, the same number is written as $(2676) {9}$ and ends in the digit $6$. for how many values of $b$ does the base-$b$-representation of $2013$ end in the digit $3$?
There are no values of $b$ for which the base-$b$-representation of $2013$ ends in the digit $3$. For a number to end in the digit $3$ in base-$b$ representation, it must be congruent to $3$ modulo $b$.
We can rewrite $2013$ as $2 \cdot 10^3 + 0 \cdot 10^2 + 1 \cdot 10^1 + 3 \cdot 10^0$. Now, if $2013$ is congruent to $3$ modulo $b$, it means that $2 \cdot 10^3 + 0 \cdot 10^2 + 1 \cdot 10^1 + 3 \cdot 10^0$ is also congruent to $3$ modulo $b$.
Simplifying the expression, we have $2000 + 0 + 10 + 3$. Since the base-$b$-representation is formed by multiplying each digit by the corresponding power of $b$, we can rewrite the expression as $2 \cdot b^3 + 1 \cdot b^1 + 3 \cdot b^0$. We can now observe that the constant term $3 \cdot b^0$ will always be congruent to $3$ modulo $b$. However, the other terms $2 \cdot b^3$ and $1 \cdot b^1$ will not be congruent to $3$ modulo $b$ for any positive value of $b$. Therefore, the base-$b$-representation of $2013$ cannot end in the digit $3$ for any value of $b$.
Hence, there are no values of $b$ for which the base-$b$-representation of $2013$ ends in the digit $3$.
Learn more about congruent here:
brainly.com/question/33002682
#SPJ11
What is the value of x in the equation One-third x minus two-thirds = negative 18?
–56
–52
52
56
Answer:
-52, I think
Step-by-step explanation:
What is the area of the shaded perimeter?
The required area of the shaded perimeter is given as 28 square units.
What are trigonometric equations?These are the equation that contains trigonometric operators such as sin, cos.. etc.. In algebraic operations.
Here,
As shown in the figure the shaded part is a parallelogram,
The area of the parallelogram is given as = base × height
Substitute the value in the above equation,
Area = 4sin45 × 7 / sin45 [since base / 4 = sin45 and 7 / height = sin45]
Area = 4/√2 × 7 / [1 / √2]
Area = 28
Thus, the shaded perimeter's needed area is specified as 28 square units.
Learn more about trigonometry equations here:
brainly.com/question/22624805
#SPJ1
1. Given the given cost function
C(x)=8700+440x+0.5x 2 and the demand function p(x)=1320 .
Find the production level that will maximaze profit.
2. A box with a square base and open top must have a volume of 186624 cm 3 . We wish to find the dimensions of the box that minimize the amount of material used.
First, find a formula for the surface area of the box in terms of only x , the length of one side of the square base.
[Hint: use the volume formula to express the height of the box in terms of x .]
Simplify your formula as much as possible.
3. For the given cost function
C(x)=48400+600x+x 2
find:
a) The production level that will minimize the average cost
b) The minimal average cost
4. A company's revenue from selling x units of an item is given as R=1000x−2x 2 . If sales are increasing at the rate of 55 units per day, how rapidly is revenue increasing (in dollars per day) when 210 units have been sold?
5. Suppose the Sunglasses Hut Company has a profit function given by P(q)=−0.03q 2 +5q−38 , where q is the number of thousands of pairs of sunglasses sold and produced, and P(q) is the total profit, in thousands of dollars, from selling and producing q pairs of sunglasses.
A) Find a simplified expression for the marginal profit function. (Be sure to use the proper variable in your answer.)
MP(q)=
B) How many pairs of sunglasses (in thousands) should be sold to maximize profits? (If necessary, round your answer to three decimal places.)
C) What are the actual maximum profits (in thousands) that can be expected? (If necessary, round your answer to three decimal places.)
the maximum profit, we plug q = 83.33 into the profit function: P(83.33) = -0.03(83.33)^2 + 5(83.33) - 38 = $348.89 thousand.
1. To maximize profit, we need to find the production level where revenue (p(x)*x) minus cost (C(x)) is highest. So we need to find the derivative of the profit function:
P(x) = p(x)*x - C(x) = 1320x - 8700 - 440x - 0.5x^2
P'(x) = 1320 - 440 - x = 880 - x
Setting P'(x) = 0, we get x = 880. So the production level that will maximize profit is 880 units.
2. Let x be the length of one side of the square base, and h be the height of the box. We know that volume V = x^2*h = 186624, so h = 186624/x^2. The surface area of the box is A = x^2 + 4xh = x^2 + 4x(186624/x^2) = x^2 + 746496/x.
3. To find the average cost function, we divide the cost function by the production level:
AC(x) = C(x)/x = (48400+600x+x^2)/x = 48400/x + 600 + x
To minimize AC(x), we need to find the derivative of AC(x):
AC'(x) = -48400/x^2 + 1
Setting AC'(x) = 0, we get x = sqrt(48400) = 220. To confirm that this is the minimum, we can take the second derivative:
AC''(x) = 96800/x^3 > 0, so x = 220 is indeed the production level that minimizes average cost.
To find the minimal average cost, we plug x = 220 into the AC(x) equation:
AC(220) = 48400/220 + 600 + 220 = 802.
4. We need to find the derivative of the revenue function with respect to time:
R(x) = 1000x - 2x^2
dR/dt = dR/dx * dx/dt = (1000 - 4x) * 55
When 210 units have been sold, x = 210, so dR/dt = (1000 - 4*210) * 55 = $22,000 per day.
5. The marginal profit function is the derivative of the profit function:
P(q) = -0.03q^2 + 5q - 38
MP(q) = dP/dq = -0.06q + 5
To find the quantity that maximizes profit, we need to set MP(q) = 0:
-0.06q + 5 = 0
q = 83.33 thousand pairs of sunglasses.
To find the maximum profit, we plug q = 83.33 into the profit function:
P(83.33) = -0.03(83.33)^2 + 5(83.33) - 38 = $348.89 thousand.
to learn more about the profit function click here:
https://brainly.com/question/16866047
#SPJ11
need help asap. low geometry grade
Answer:
x=9.3
Step-by-step explanation:
use SohCahToa
in this case u use cos
cos(41°)=7/x
x=7/cos(41)
x=9.275090953
x=9.3
EASY BRAINLIEST PLEASE HELP!!
-if you answer correctly ill give you brainliest which will give you 100pts-
(if you put links or don't answer accordingly im reporting your account)
Answer:
The parallelogram ABCD is what makes the parallelogram.
The proof given was AC and BD bisect each other.
Bisect means to divide into 2 parts. AC and BD both divide into 2 parts with each other.
But the lines need to be in 2 equal parts.
So if we were to divide it in that case, it would be able to bisect.
This is given proof that parallelogram ABCD is a parallelogram.
Answer:
Solution given:
ABCD is a parallegram.
In ∆ADE and ∆BCE
AD=BC[corresponding side of parallelogram are equal]
<AED=<BEC[ vertically opposite angle]
<ADE=<EBC[alternate angle]
so
∆ADE are congruent to ∆BCE by S.A.A. axiom
we have
DE=BE[corresponding side of congruent triangle are equal]
Similarly
AE=CE
we have
AC=BD
AE+CE=BE+DE
2AE=2BE
AE=BE
so.we can rightly say that AC and BD bisect each other.
. You deposit $200 each month into an account earning 3% interest compounded monthly for 30 years. How much total interest will you earn in 30 years?
The total interest you will earn in 30 years is approximately $241.61.
To calculate the total interest earned in 30 years, we need to use the formula for compound interest:
A = P(1 + r/n)^(nt) - P
Where:
A = the future value of the investment
P = the principal amount (initial deposit)
r = the annual interest rate (in decimal form)
n = the number of times the interest is compounded per year
t = the number of years
In this case, the principal amount is $200, the annual interest rate is 3% (or 0.03 as a decimal), the interest is compounded monthly (so n = 12), and the time period is 30 years (so t = 30).
Plugging in these values into the formula:
A = 200(1 + 0.03/12)^(12*30) - 200
Now we can simplify and calculate:
A = 200(1.0025)^(360) - 200
A = 200(2.208040033) - 200
A ≈ 441.6080066 - 200
A ≈ 241.6080066
For more such questions on approximately
https://brainly.com/question/29985480
#SPJ8
Find m<1 and m<2. Justify your answer.
Answer:
M<1 80 by the corresponding angles postulate
Step-by-step explanation:
Please help I need to finish this in 2 days
The angle subtended at the center of the arc is 102⁰
What is the length of the arc?Recall that to find the length of an arc on a circle, we can use the formula L = r *, where r is the radius of the circle, is the central angle, and is the angle between the ends of the arc. If the central angle is measured in degrees, we can use the formula L = r *, where r is the radius of the circle, is the central angle, and is the angle between the ends of the arc.
Lenght of arc = A/360 *2пr
A = angle at center = ?
п = 22/7 r = radius = 840 feet
⇒1500 = A/360 2 *22/7 * 840
1500 = 36960A/2520
3780000= 36960A
making A the subject we have
3780000/36960 = A
A = 102.27
A= 102⁰
The angle is 102⁰
Learn more about length of arc on https://brainly.com/question/31762064
#SPJ1
There are 10 sweets in a bag. 4 are red, 2 are green, 3 are yellow and 1 is purple. A sweet is chosen at random from the bag. Here is a probability scale:
a) which letter shows the probability of choosing a yellow sweet?.
Answer: 3 / 10
Step-by-step explanation:
It's a 30% chance.
Answer:
I dont know the probability scale letters, but the probability of getting a yellow sweet would be 3/10, 0.3, or 30% (they all say the same thing just different ways)
Hope this helps :)
Step-by-step explanation:
Bus Stop #2:
initial value: 9 miles
rate of change: 0.5 miles each minute
Friend’s House:
A 2-column table with 3 rows. Column 1 is labeled Time (minutes) with entries 3, 6, 9. Column 2 is labeled Distance (miles) with entries 4, 6, 8.
Seth can also get dropped off at Bus Stop #2, which is closer to his school.
Which will get him to school faster? (School is 17 miles away.)
Answer:
✔ bus stop #2, by 6.5 minutes
Step-by-step explanation:
Answer:
✔ bus stop #2, by 6.5 minutes
Step-by-step explanation:
PLS HELP MEEEEEE ( I don’t need to know the first one )
Answer:
k=27
Step-by-step explanation:
Answer:
direct variation is y = kx
k is the slope of the line y = mx + b
i don't know that last one, sorry
Step-by-step explanation:
The iterative process below can be used to find 2 approximate solutions to x³ - 5x² - 11 = 0 to 2 d.p. Starting with x = 5, use the iterative process to find an approximate solution to x³ - 5x² - 11 = 0. Give your answer to 2 d.p. X Step 1: Start with a value of x Step 2: Find the value of 5+ TAL 11 x² Step 3: Round your answer to Step 2 and the value of x to 2 d.p. If they are the same, then stop. You have found an approximate solution. If not, then go back to Step 1, using your exact answer to Step 2 as the new value for x.
An approximate solution to x³ - 5x² - 11 = 0, rounded to 2 decimal places, is x ≈ 2.76.
How to calculate the valueUse the equation derived from the iterative process:
xᵢ₊₁ = xᵢ - (f(xᵢ) / f'(xᵢ))
Calculate f(xᵢ):
f(xᵢ) = xᵢ³ - 5xᵢ² - 11
Calculate f'(xᵢ):
f'(xᵢ) = 3xᵢ² - 10xᵢ
Substitute the values of xᵢ, f(xᵢ), and f'(xᵢ) into the iterative equation and calculate xᵢ₊₁.
Let's perform the calculations:
For x = 5:
f(x) = 5³ - 5(5)² - 11 = 69
f'(x) = 3(5)² - 10(5) = 25
Using the iterative equation:
x₁ = 5 - (69 / 25)
≈ 2.76
Therefore, an approximate solution to x³ - 5x² - 11 = 0, rounded to 2 decimal places, is x ≈ 2.76.
Learn more about equations on
https://brainly.com/question/2972832
#SPJ1
HELP ASAP THIS IS DUE IN AN HOUR ILL MARK U BRIANLY
Answer:
5.8 plus 5.1 plus 2
final answer is 11.9
Can anyone help me figure out what AP:PQ is. I keep getting stuck at y=-12y which can’t go anywhere…
Answer:
AP : PQ = 4 : 7
Step-by-step explanation:
A, P and Q lie on a straight vertical line with
AP = 4 units and AQ = 7 units, then
AP : PQ = 4 : 7
the tables randomly assigned to the smiley face or control condition.the tables randomly assigned to the smiley face or control condition. b the servers randomly assigned to the smiley face or control condition.the servers randomly assigned to the smiley face or control condition. c the two conditions: smiley face and control.the two conditions: smiley face and control. d the restaurants that were involved in this study.
The answer is c) the two conditions: smiley face and control. This study involved testing the effect of smiley faces on tipping behavior in restaurants.
The two conditions being tested were the presence of a smiley face on the customer's bill and the absence of a smiley face on the customer's bill (control condition).
In this study, researchers randomly assigned customers to either the smiley face or control condition and measured the amount of tip left by each customer.
The results showed that customers who received a bill with a smiley face left a significantly higher tip compared to those who received a bill without a smiley face. The study suggests that simple gestures such as adding a smiley face to a bill can have a positive effect on customer behavior in restaurants.
Complete Question:
what are the cases involved in this study? responses
a) the tables randomly assigned to the smiley face or control condition. the tables randomly assigned to the smiley face or control condition.
b) the servers randomly assigned to the smiley face or control condition. the servers randomly assigned to the smiley face or control condition.
c ) the two conditions: smiley face and control. the two conditions: smiley face and control. d the restaurants that were involved in this study.
Learn more about Conditions:
brainly.com/question/30267509
#SPJ11
the length of a rectangle is 1 yd more than twice the width, and the area of the rectangle is 45 yd2. find the dimensions of the rectangle. length : yd width : yd
On solving the provided question, we can say that in rectangle W = 4.5 yd since we are unable to have a negative length. L = 2W + 1 = 10 yd
What is rectangle?A rectangle in Euclidean plane geometry is a quadrilateral with four right angles. You might also describe it as follows: a quadrilateral that is equiangular, which indicates that all of its angles are equal. The parallelogram might also have a straight angle. Squares are rectangles with four equally sized sides. A quadrilateral of the shape of a rectangle has four 90-degree vertices and equal parallel sides. As a result, it is sometimes referred to as an equirectangular rectangle. Because its opposite sides are equal and parallel, a rectangle is also known as a parallelogram.
length rectangle is 1 yd more than twice the width: L = 2*W + 1
e area of the rectangle
\(45 = L*W\\45 = (2W + 1)*W\\45 = 2W2 + W\\0 = 2W2 + W - 45\\\)
Factors
x\((2W-9)(W+5)\\W = 4.5 and -5\)
W = 4.5 yd since we are unable to have a negative length. L = 2W + 1 = 10 yd
To know more about rectangle visit:
https://brainly.com/question/29123947
#SPJ4
HELP URGENT!!!!! WILL MARK BRAINLIEST!!!!
Which equation best fits the graph given?
a. f(x)=(x-1)^3 (x+2)^2
b. f(x)=(x+1)^2(x-2)^3
c. f(x)=(x-1)^2 (x+2)^3
d. f(x)=(x+1)^3(x-2)^2
e. None of the above
Briefly explain how you decided on the equation.
Answer:
D
Step-by-step explanation:
first find the zeros on the graph and their multiplicity.
zeros: -1, 2
mult: 3, 2
the equation that fits is f(x)=(x+1)^3(x-2)^2
Answer A B C or D and explain
Answer:
i believe its just A
Step-by-step explanation:
Explanation:
Relation I has each item in the domain map to exactly one item in the range. So any input leads to exactly one output. Therefore, we have a function here.
In contrast, relation II has the input x = 0 lead to multiple outputs (namely y = 8 and y = -1 at the same time). This indicates we do not have a function here.
Relation III is the same as as the previous one. This time the input x = -2 maps to both y = 1 and y = -7 at the same time. We don't have a function here either.
If you wanted, you can plot these points on the xy plane to see that relations II and III fail the vertical line test. You would need to plot each relation separately.
help please. decimals, fractions.
Answer:
8/2
Step-by-step explanation:
7.5 is 8 halves and its over 2 because that's how many halves there are